1
$\begingroup$

Hi there, I have been studying the following set (in order to investigate the average of products of Ramanujan sums with some weights):

$$ A=\lbrace (n,m) \in \mathbb{Z}/q\mathbb{Z} \times \mathbb{Z}/q\mathbb{Z} \mid \text{ }nm\equiv a \mod{q},\text{ } n \equiv 1 \mod{d_{1}}, \text{ } m \equiv 1 \mod{d_{2}} \rbrace $$

where $q$ is any positive integer, $d_{1}$ and $d_{2}$ are proper divisors of $q$ and $(a,q)=1$.

For the case $a=1$, using a bit algebra I have deduced the elementary formula

$$\sum_{\substack{n_{1}n_{2}\equiv 1\mod{q}, \\\ n_{1} \equiv 1 \mod{d_{1}}, \\\ n_{2} \equiv 1 \mod{d_{2}}}}=\frac{\phi(q)}{\mathrm{lcm}(\phi(d_{1}),\phi(d_{2}))}.$$

Is it possible to derive a formula for $|A|$?

$\endgroup$
5
  • 5
    $\begingroup$ The sum of WHAT, and what exactly is your question? $\endgroup$
    – Seva
    Feb 11, 2013 at 16:17
  • $\begingroup$ Actually we want to count a set, which we abbreviate by summing 1s and putting constraints as indexes. $\endgroup$ Feb 11, 2013 at 17:49
  • $\begingroup$ There are no 1s in your question. And what is $k$ in the right hand side of the second formula? $\endgroup$ Feb 11, 2013 at 17:53
  • 3
    $\begingroup$ Then the WHAT is 1, and you need to put it in the expression. Gerhard "Or Suffer The Community Wrath" Paseman, 2013.02.11 $\endgroup$ Feb 11, 2013 at 17:55
  • $\begingroup$ Question also posted to m.se, math.stackexchange.com/questions/299752/… $\endgroup$ Feb 12, 2013 at 0:05

1 Answer 1

2
$\begingroup$

I believe the formula $|A|=\frac{\varphi(q)}{\mathrm{lcm}(\varphi(d_1),\varphi(d_2))}$ is not quite correct. In particular, for $q=15$, $d_1 = 3$, $d_2=5$, $a=1$, this formula gives $|A|=2$, while, in fact, $A = \{ (1,1) \}$ with $|A|=1$. Below I derive a correct formula.

First, it is clear that if $a\not\equiv1\pmod{\gcd(d_1,d_2)}$, then $|A|=0$. For the rest assume that $a\equiv1\pmod{\gcd(d_1,d_2)}$.

Let $p$ be a prime dividing $q$ and $t=\nu_p(q)>0$ (i.e., $t$ is the valuation of $q$ w.r.t. $p$) and $s_1 = \nu_p(d_1)$, $s_2 = \nu_p(d_2)$. It is easy to see that the number of elements modulo $p^t$ in $A$ is indeed $$\frac{\varphi(p^t)}{\mathrm{lcm}(\varphi(p^{s_1}),\varphi(p^{s_2}))} = \begin{cases} (p-1)p^{t-1},& \text{if}\ s_1=s_2=0\\\\ p^{t-\max\{s_1,s_2\}},&\text{otherwise}. \end{cases}$$

Now, for any $a$ such that $\gcd(a,q)=1$ and $a\equiv1\pmod{\gcd(d_1,d_2)}$, we have (thanks to CRT) $$ |A| = \prod_{p|q} \frac{\varphi(p^{\nu_p(q)})}{\mathrm{lcm}(\varphi(p^{\nu_p(d_1)}),\varphi(p^{\nu_p(d_2)}))} = \frac{\varphi(q)}{\prod_{p|q} \mathrm{lcm}(\varphi(p^{\nu_p(d_1)}),\varphi(p^{\nu_p(d_2)}))}.$$ Notice that this product does not collapse into $\frac{\varphi(q)}{\mathrm{lcm}(\varphi(d_1),\varphi(d_2))}$.

$\endgroup$

Your Answer

By clicking “Post Your Answer”, you agree to our terms of service and acknowledge that you have read and understand our privacy policy and code of conduct.

Not the answer you're looking for? Browse other questions tagged or ask your own question.